LSAT and Law School Admissions Forum

Get expert LSAT preparation and law school admissions advice from PowerScore Test Preparation.

 Administrator
PowerScore Staff
  • PowerScore Staff
  • Posts: 8917
  • Joined: Feb 02, 2011
|
#80568
Complete Question Explanation

Strengthen-PR. The correct answer choice is (D).

Answer choice (A):

Answer choice (B):

Answer choice (C):

Answer choice (D): This is the correct answer choice.

Answer choice (E):

This explanation is still in progress. Please post any questions below!
 lsatryan
  • Posts: 8
  • Joined: Nov 09, 2020
|
#82633
Hello, would you be able to clarify why E is incorrect? Thank you
User avatar
 KelseyWoods
PowerScore Staff
  • PowerScore Staff
  • Posts: 1079
  • Joined: Jun 26, 2013
|
#82680
Hi Ryan!

This is a Strengthen-Principle question. Let's first break down the argument in the stimulus:

Premise: Pollution from automobiles is a major contributor to global warming
Premise: Global warming is causing inhabitants of arctic villages to relocate
Conclusion: Automotive industry should be required to help pay for the relocation

When you're looking for a principle to help strengthen an argument, you're basically looking for a principle that says that if the premises are true, then the conclusion is true. In conditional terms, that means we need a principle that matches:

Premise :arrow: Conclusion
Auto industry is causing arctic villagers to relocate :arrow: Auto industry should help pay for relocation

Answer choice (D) states: "Any industry manufacturing a product whose use contributes to costly damage for others should be liable for any damage generated by that product's use." The word "any" is a sufficient indicator, so we can diagram that statement as:

Industry manufacturing a product that causes damage :arrow: Industry should be held liable for damage

That maps on really well with our argument above. If any industry manufacturing a product that causes damage should be held liable for that damage, then if the auto industry is causing damage to arctic villages, it should be held liable for that damage.

Now check out answer choice (E): "An industry that contributes to global warming should be required to help pay for resulting damage to specific communities only if it has a general obligation to help pay for all damage produced by global warming." The phase "only if" is a necessary condition indicator, so we would diagram that statement as:

Industry that contributes to global warming should help pay for global warming damage :arrow: Industry that contributes to global warming has a general obligation to help pay for all damage produced by global warming

This answer choice puts the conclusion of the stimulus argument (that the auto industry should help pay) in the sufficient condition. For the principle to match, we need the conclusion of the argument in the necessary condition. The conclusion is what we're trying to prove based on the premises. So, again, we need the principle to basically say that if the premises are true, then the conclusion is true. We can't do much with a principle that puts the conclusion in the sufficient condition, because we don't know whether or not the conclusion is true. We accept the premises as true and we're looking for a principle that says that the conclusion follows from those premises. But since we do not know for sure whether the conclusion is true anything that starts with "if the conclusion is true..." is not going to get us anywhere.

Hope this helps!

Best,
Kelsey

Get the most out of your LSAT Prep Plus subscription.

Analyze and track your performance with our Testing and Analytics Package.